How can K be on Sunday?
If you put K on Sunday, then K is on saturday which forces a G on Monday and Friday. P is on Tues...
Caden 02:29PM
  • November 2018 LSAT
  • SEC2
  • Q23
1
Reply
Explanation
Can you explain this answer?
mmanetti 02:20PM
  • November 2018 LSAT
  • SEC2
  • Q4
1
Reply
November 2018 SEC 2 QX 6
Why is B incorrect, if K&O lecture same day KOJ LMN , we would still be having JK on the same d...
Bola on October 6 at 04:33PM
  • November 2018 LSAT
  • SEC2
  • Q6
1
Reply
How is E wrong??
Explain
Tyler808 on August 29, 2022
  • November 2018 LSAT
  • SEC2
  • Q13
1
Reply
Why not 3?
JRN RVN RJN?
Olivia-Salem on April 16, 2022
  • November 2018 LSAT
  • SEC2
  • Q11
2
Replies
Why isn’t it J X M
J X M results with T- JLK F-MNO
Olivia-Salem on April 16, 2022
  • November 2018 LSAT
  • SEC2
  • Q6
1
Reply
Why can’t Navarro go on Thursday at 2? Thurs - ...
Why can’t Navarro go on Thursday at 2? Thurs - M N L Friday - J O K?
Olivia-Salem on April 16, 2022
  • November 2018 LSAT
  • SEC2
  • Q4
1
Reply
Why only two?
After diagramming this, I had J,R and N as possibilities for the second slot. Why is this not the...
Stefaniggorman on April 15, 2022
  • November 2018 LSAT
  • SEC2
  • Q11
3
Replies
Why is a incorrect?
Hi, I don't understand why a is incorrect. I set up a couple of scenarios. Th: J, O/K, K/O F: L...
Abigail-Okereke on March 9, 2022
  • November 2018 LSAT
  • SEC2
  • Q2
1
Reply
Can you please post a video for the game?
I read through your game set-up, but am still fairly confused. Can you please post a video on thi...
Stefaniggorman on October 21, 2021
  • November 2018 LSAT
  • SEC2
  • Q13
3
Replies
Why is the right answer A? Couldn't J be in 6th...
Couldn't J be in 6th and K still be 4th? If it goes R,V,N,K,S,J or V,R,N,K,S,J or R,N,V,K,S,J the...
greatdane28 on August 25, 2021
  • November 2018 LSAT
  • SEC2
  • Q9
4
Replies
Game setup please?
Can someone please explain the setup of this game?
jewan on March 13, 2021
  • November 2018 LSAT
  • SEC2
  • Q13
5
Replies
Why is B the correct answer?
If M is at 1 that means J is in the other 1 spot. Lewis can only be on Friday if he is at 1. What...
Matthew-Beecham on October 28, 2020
  • November 2018 LSAT
  • SEC2
  • Q2
2
Replies
Why is C incorrect?
If N and O are together, and MN are on the same day, then one day will be MNO, which would force ...
makda.mehari on October 15, 2020
  • November 2018 LSAT
  • SEC2
  • Q6
1
Reply
Ans choice D
Hi, i read your explanation in the previous thread. I used the same logic, then why is D wrong. J...
ankita96 on August 10, 2020
  • November 2018 LSAT
  • SEC2
  • Q6
3
Replies
How is the Answer D? Confused, please help
looking at the possible 4 options, we would be in the #3 option? Which is the only option where G...
aseikhon11 on July 30, 2020
  • November 2018 LSAT
  • SEC2
  • Q17
1
Reply
Contrapositive of last rule
Hi, I just want to make sure I properly understood the last rule. I wrote the contrapositiv...
dpenn on July 25, 2020
  • November 2018 LSAT
  • SEC2
  • Q1
1
Reply
Please explain between A & B
Hey!! Can you please please explain why the correct answer choice is A instead of B. I already re...
Maria-Marin on July 14, 2020
  • November 2018 LSAT
  • SEC2
  • Q9
1
Reply
Answer choice A
Hey!! could you please explain me why Answer choice A could not be correct?
Maria-Marin on July 14, 2020
  • November 2018 LSAT
  • SEC2
  • Q16
1
Reply
Answer choice A
Heyyy could you please tell me why A is not right ?
Maria-Marin on July 14, 2020
  • November 2018 LSAT
  • SEC2
  • Q22
1
Reply